Menu Close

i-1-1-p-i-p-i-is-i-th-prime-diverges-i-1-1-s-i-s-i-is-i-th-whole-square-converges-Why-Even-though-it-appears-that-we-have-more-whole-squares-than-primes-




Question Number 3546 by prakash jain last updated on 15/Dec/15
Σ_(i=1) ^∞ (1/p_i ), p_i  is i^(th)  prime diverges.  Σ_(i=1) ^∞ (1/s_i ), s_i  is i^(th)  whole square converges.  Why?  Even though it appears that we have more  whole squares than primes.
$$\underset{{i}=\mathrm{1}} {\overset{\infty} {\sum}}\frac{\mathrm{1}}{{p}_{{i}} },\:{p}_{{i}} \:{is}\:{i}^{{th}} \:{prime}\:{diverges}. \\ $$$$\underset{{i}=\mathrm{1}} {\overset{\infty} {\sum}}\frac{\mathrm{1}}{{s}_{{i}} },\:{s}_{{i}} \:{is}\:{i}^{{th}} \:{whole}\:{square}\:{converges}. \\ $$$$\mathrm{Why}? \\ $$$$\mathrm{Even}\:\mathrm{though}\:\mathrm{it}\:\mathrm{appears}\:\mathrm{that}\:\mathrm{we}\:\mathrm{have}\:\mathrm{more} \\ $$$$\mathrm{whole}\:\mathrm{squares}\:\mathrm{than}\:\mathrm{primes}. \\ $$
Commented by Yozzii last updated on 15/Dec/15
Yes.
$${Yes}. \\ $$
Commented by prakash jain last updated on 15/Dec/15
Thanks. It answers the question that  the sum of inverse of prime number can  be divergent if (1/n^2 ) is convergent.
$$\mathrm{Thanks}.\:\mathrm{It}\:\mathrm{answers}\:\mathrm{the}\:\mathrm{question}\:\mathrm{that} \\ $$$$\mathrm{the}\:\mathrm{sum}\:\mathrm{of}\:\mathrm{inverse}\:\mathrm{of}\:\mathrm{prime}\:\mathrm{number}\:\mathrm{can} \\ $$$$\mathrm{be}\:\mathrm{divergent}\:\mathrm{if}\:\frac{\mathrm{1}}{{n}^{\mathrm{2}} }\:\mathrm{is}\:\mathrm{convergent}. \\ $$
Commented by Filup last updated on 15/Dec/15
Σ_(i=1) ^∞ (1/s_i )=Σ_(i=1) ^∞ (1/( (√s_i ))) (1/( (√s_i )))           (√s_i )∈Z  p_i  has no factors
$$\underset{{i}=\mathrm{1}} {\overset{\infty} {\sum}}\frac{\mathrm{1}}{{s}_{{i}} }=\underset{{i}=\mathrm{1}} {\overset{\infty} {\sum}}\frac{\mathrm{1}}{\:\sqrt{{s}_{{i}} }}\:\frac{\mathrm{1}}{\:\sqrt{{s}_{{i}} }}\:\:\:\:\:\:\:\:\:\:\:\sqrt{{s}_{{i}} }\in\mathbb{Z} \\ $$$${p}_{{i}} \:{has}\:{no}\:{factors} \\ $$
Commented by Yozzii last updated on 15/Dec/15
(1/3)>(1/9)      (1/7)>(1/(25))  (1/2)>(1/4)       (1/5)>(1/(16))  ∴ (1/p_i )>(1/s_i ) ∀i∈N  ∴ Σ_(i=1) ^∞ (1/p_i )>Σ_(i=1) ^∞ (1/s_i )−(1/1)  1+Σ_(i=1) ^∞ (1/p_i )>Σ_(i=1) ^∞ (1/s_i )  s_i =i^2    i∈Z^+    ∴ Σ_(i=1) ^∞ (1/s_i )=Σ_(i=1) ^∞ (1/i^2 ). p series with p=2.  Since p>1, the series is convergent.
$$\frac{\mathrm{1}}{\mathrm{3}}>\frac{\mathrm{1}}{\mathrm{9}}\:\:\:\:\:\:\frac{\mathrm{1}}{\mathrm{7}}>\frac{\mathrm{1}}{\mathrm{25}} \\ $$$$\frac{\mathrm{1}}{\mathrm{2}}>\frac{\mathrm{1}}{\mathrm{4}}\:\:\:\:\:\:\:\frac{\mathrm{1}}{\mathrm{5}}>\frac{\mathrm{1}}{\mathrm{16}} \\ $$$$\therefore\:\frac{\mathrm{1}}{{p}_{{i}} }>\frac{\mathrm{1}}{{s}_{{i}} }\:\forall{i}\in\mathbb{N} \\ $$$$\therefore\:\underset{{i}=\mathrm{1}} {\overset{\infty} {\sum}}\frac{\mathrm{1}}{{p}_{{i}} }>\underset{{i}=\mathrm{1}} {\overset{\infty} {\sum}}\frac{\mathrm{1}}{{s}_{{i}} }−\frac{\mathrm{1}}{\mathrm{1}} \\ $$$$\mathrm{1}+\underset{{i}=\mathrm{1}} {\overset{\infty} {\sum}}\frac{\mathrm{1}}{{p}_{{i}} }>\underset{{i}=\mathrm{1}} {\overset{\infty} {\sum}}\frac{\mathrm{1}}{{s}_{{i}} } \\ $$$${s}_{{i}} ={i}^{\mathrm{2}} \:\:\:{i}\in\mathbb{Z}^{+} \: \\ $$$$\therefore\:\underset{{i}=\mathrm{1}} {\overset{\infty} {\sum}}\frac{\mathrm{1}}{{s}_{{i}} }=\underset{{i}=\mathrm{1}} {\overset{\infty} {\sum}}\frac{\mathrm{1}}{{i}^{\mathrm{2}} }.\:{p}\:{series}\:{with}\:{p}=\mathrm{2}. \\ $$$${Since}\:{p}>\mathrm{1},\:{the}\:{series}\:{is}\:{convergent}. \\ $$

Leave a Reply

Your email address will not be published. Required fields are marked *